¿Todo operador hermitiano representa una cantidad medible?

En la mecánica cuántica, los observables están representados por el operador hermitiano. Pero, ¿todo operador hermitiano representa un observable? Si no, ¿cómo sabemos si un operador hermitiano representa observable o no? ¿Cuál es la definición precisa del término "observable"?

Relacionado (ver también los comentarios allí): physics.stackexchange.com/q/54603
Posibles duplicados: physics.stackexchange.com/q/27038/2451 y enlaces allí.

Respuestas (1)

Dado un sistema cuántico con espacio de Hilbert asociado H , el conjunto de todos los operadores acotados autoadjuntos es B ( H ) sa . En general, sólo un pequeño subconjunto de B ( H ) sa representará operadores físicamente observables. Para sistemas de dimensión infinita, B ( H ) sa es enorme y no hay esperanza de encontrar experimentos para todos sus miembros; incluso en sistemas de dimensión finita es muy difícil encontrar esquemas experimentales sensibles incluso a una base de espacio vectorial para B ( H ) sa .

El enfoque físico para esto es comenzar con un conjunto finito de operadores que sabe que puede medir. Para una sola partícula libre, por ejemplo, tomarías la posición y el momento; para un conjunto finito de giros, tomarías todas sus matrices de Pauli. Luego formas el conjunto A de todos los operadores que pueden formarse a partir de ellos mediante productos y combinaciones lineales, que tiene la estructura de un C álgebra, y ese es su conjunto de observables físicos. los C el álgebra misma es la descripción realmente fundamental del sistema ; el espacio de Hilbert es simplemente una representación posible.

En este formalismo, los estados son funcionales en A : son funciones

ρ : A C
que toman un observable y dan su valor medido (o valor medido probable, etc.) en ese estado. (En una representación espacial de Hilbert, cada funcional está asociado con una matriz de densidad ρ ^ , un operador positivo de clase traza tal que ρ ( A ) = Tr ( ρ ^ A ^ ) por A ^ el operador espacial de Hilbert asociado con un arbitrario A A .

Editar: como joshphysics y WetSavannaAnimal señalan correctamente, esto funciona como se indica solo para operadores acotados y no para operadores ilimitados como posición o energía. Me temo que no sé lo suficientemente bien cómo se extiende esto a esa clase de operadores: eso necesita a alguien con habilidades de análisis funcional mucho más fuertes que las mías.

@dj_mummy Debería pensar que la posibilidad de observables adicionales nunca se puede descartar de manera concluyente. Sin embargo, como serían observables completamente cuánticos, no ayudarían a resolver las paradojas EPR: son locales y, por lo tanto, dentro del tratamiento de Bell, o entrelazados, en cuyo caso se rompe la suposición de localidad.
@EmilioPisanty Estoy confundido. En los cursos estándar de mecánica cuántica, generalmente consideramos que ciertos operadores autoadjuntos no acotados son observables, pero presumiblemente B ( H ) s a no contiene estas bestias. ¿Qué me estoy perdiendo?
@Emilio: a "cada uno de esos funcionales está asociado con una matriz de densidad": esto es cierto solo para estados normales, consulte ncatlab.org/nlab/show/state+on+an+operator+algebra
@joshphysics: cada operador normal (tal vez ilimitado) se puede escribir como una integral sobre los operadores de proyección (que están limitados). Entonces, si conoce los valores esperados de los operadores de proyección, también conoce el valor esperado del operador normal.
@jjcale ¿Tiene una referencia para esto? No recuerdo que la descomposición espectral para operadores ilimitados sea tan simple.
@joshphysics Los proyectores espectrales pueden ser difíciles de construir con su operador original, pero siempre están limitados. (Por un lado, su espectro es { 0 , 1 } .) Si conoce los valores esperados Tr ( ρ ^ Π ^ [ X 1 , X 2 ] ) entonces puedes usarlos para integrar para Tr ( ρ ^ A ^ ) .
Emilio y @joshphysics . Sería genial obtener una referencia: ¿me estoy perdiendo algo (¡probablemente!)? No está resolviendo un operador en su suma de peso ilimitado de descomposición espectral de proyectores siempre limitados, por lo que su comentario parece un poco como una pregunta de principio. No dudo que tengas razón (¡he visto un buen número de tus publicaciones!) - es solo que mi concepción (y probablemente la de joshphysics) de la teoría espectral es algo que obtiene muchos detalles complicados cuando deambular por las tierras ilimitadas del operador. ¡Me encantaría ver mejores versiones y parece que las has visto!
@EmilioPisanty ¿Es que prácticamente no hay esperanza de encontrar experimentos para medir todos los operadores hermitianos o incluso, fundamentalmente , no todos los operadores hermitianos son observables? Estoy descuidando las consideraciones de redundancia de calibre (o, digamos, estoy identificando todos los espacios de Hilbert transformados por calibre como un solo espacio físico de Hilbert). Si no todos los operadores hermitianos son observables (en un sentido fundamental), ¿no proporcionaría un (conjunto de) bases preferidas para el espacio de Hilbert? No veo una contradicción si la respuesta a esta pregunta es sí, ¡pero "se siente" incómodo! ¡Gracias!